Relativistic E and B fields of an infinitely long wire

In summary, the problem at hand involves an infinitely long and thin straight wire with a constant charge density and moving at a constant relativistic speed perpendicular and parallel to its axis. The task is to determine the electric and magnetic fields in the entire space and the charge density and current density that generate those fields. The electrostatic potential for a similar situation is given as a tip. The solution involves calculating the electric and magnetic fields in the frame of the moving wire and then performing a Lorentz transformation to find the fields in the non-moving frame. However, there may be complications due to the orientation of the rod changing as the frame changes.
  • #1
JulienB
408
12

Homework Statement



Hi everybody! I have the following problem to solve:

An infinitely long and thin straight wire carries a constant charge density ##\lambda## and moves at a constant relativistic speed ##\vec{v}## perpendicularly (##\beta##) and parallel (##\alpha##) to its axis.
a) Determine the ##\vec{E}## and ##\vec{B}## fields in the whole space.
b) Through which charge density ##\rho## and current density ##\vec{j}## are those fields generated?
Tip: previously we had the electrostatic potential ##\phi = - \lambda \ln(x^2+y^2)## for an infinitely long straight wire aligned with the ##z##-axis and with charge density ##\lambda##.

Homework Equations



##\vec{E} = -\nabla \phi - \partial_t \vec{A}##
##\vec{A} = \frac{\mu_0}{4 \pi} \int d^3x'\ \frac{\vec{j} (\vec{x}', t_r)}{|\vec{x} - \vec{x}'|}##
where ##t_r = t - \frac{|\vec{x} - \vec{x}'|}{c}##
##j_{\mu} = (j^0, j^i) = (c \rho, \rho \dot{\vec{x}})##

The Attempt at a Solution



Not sure what "parallel" and "perpendicular" here mean. Because of the given potential, I assumed that the wire is moving in the ##x##-direction at speed ##\alpha## and in the ##y##-direction at speed ##\beta##. Would you agree to this interpretation? It could also be that the wire moves in the ##z##-direction though ("parallel" to its axis), but then I imagine that the scalar potential should not be independent of ##z##. Is that right?

If my interpretation is correct I can calculate ##-\nabla \phi## which yields

##-\nabla \phi = \frac{2 \lambda}{ x^2+y^2} (x,y,0) = \frac{2 \lambda t}{t^2 (\alpha^2 + \beta^2)} (\alpha, \beta, 0)##

where I set ##x=\alpha t## and ##y=\beta t##. I am now not sure how to proceed regarding ##\vec{A}## since we didn't study the retarded time/potential yet (I could give it a try though). Should I try to calculate ##t_r## or is there another method I can use?Thanks a lot in advance for your suggestions.Julien.
 
Physics news on Phys.org
  • #2
Hi again! Actually I got another idea which sounds better to me even though I still don't manage to make it work.

I first calculate ##\vec{E}'(\vec{x}')## in the frame ##\Sigma'## of the moving wire so that I find myself in an electrostatics situation, therefore allowing me to calculate ##\vec{E}'(\vec{x}') = - \nabla \phi (\vec{x})## and having ##\vec{B}=0## since there is no current in that inertial frame. Then I'd perform a Lorentz transformation on both fields so that I retrieve the electric and magnetic fields in the not moving frame ##\Sigma##. Would that make sense?

Thanks a lot in advance for your answers.Julien.
 
  • #3
JulienB said:
I first calculate ##\vec{E}'(\vec{x}')## in the frame ##\Sigma'## of the moving wire so that I find myself in an electrostatics situation, therefore allowing me to calculate ##\vec{E}'(\vec{x}') = - \nabla \phi (\vec{x})## and having ##\vec{B}=0## since there is no current in that inertial frame. Then I'd perform a Lorentz transformation on both fields so that I retrieve the electric and magnetic fields in the not moving frame ##\Sigma##. Would that make sense?
Hi, Julien. That sounds like a good way to go.
 
  • #4
Hi @TSny and thanks for your answer. I give it a try:

I will redefine the velocity components because I want to keep ##\beta## for the Lorentz transformation. Therefore, the velocity is simply ##\vec{v}=(0, v_y, v_z)##. The frame of the moving wire is ##\Sigma'## while the non moving frame is ##\Sigma##. Another way to describe the latter is to say that it moves with velocity ##-\vec{v}## relative to the wire.

  • In ##\Sigma'##:
##\vec{v'}=0 \implies \begin{cases} \vec{E}' (\vec{x}') = - \nabla \phi' (\vec{x}') = \frac{2 \lambda}{x'^2+y'^2} (x',y',0) \\ \vec{B}'(\vec{x}')= 0 \end{cases}##

  • In ##\Sigma##:
##\vec{v} = (0,-v_y,-v_z) \implies x'=x, y'=\gamma(y+v_y t), z' = \gamma v_z t##
##\implies \vec{E}(\vec{x}) = \gamma (\vec{E}'(\vec{x}') + \frac{1}{c} (\vec{v} \times \vec{B}'(\vec{x}')) + \frac{\gamma^2}{c^2(1+\gamma)} (\vec{v} \cdot \vec{E}'(\vec{x}')) \vec{E}'(\vec{x}')##
##= \frac{2 \lambda \gamma}{x^2+\gamma^2(y+v_y t)^2} \bigg(1 + \frac{\gamma^2}{c^2(1+\gamma)} \frac{-v_y (y+ v_y t) - v_z^2 t}{x^2+\gamma^2(y+v_y t)^2} \bigg) (x, \gamma(y+v_y t), \gamma v_z t)##.

Well well that looks strange, especially since I was expecting that ##E_x'(\vec{x}') = E_x (\vec{x})##.. Am I doing something wrong with the Lorentz transformation? Did I transform for example the ##z##-component in the right way?Thank you very much in advance.Julien.
 
  • #5
Your Lorentz transformation equations for ##y'## and ##z'## don't appear to be correct.
https://en.wikipedia.org/wiki/Lorentz_transformation#Vector_transformations

Also, unless I'm mistaken, there is a complication due to the fact that the orientation of the rod changes as you change frames. You have the rod oriented along the z'-axis in the primed frame. However, in that case, the rod will generally not be oriented along the z-axis in the unprimed frame. Thus, your boost velocity component ##v_z## will not be parallel to the rod in the unprimed frame. Likewise, ##v_y## will not be perpendicular to the rod in the unprimed frame. So, ##v_z## and ##v_y## would not represent the quantities ##\alpha## and ##\beta## given in the problem statement.

At first, I thought the problem might be asking you to treat two cases separately:
(1) boost velocity perpendicular to rod
(2) boost velocity parallel to rod

That would be much easier than the case where the boost velocity has a nonzero component perpendicular to the rod and a nonzero component parallel to the rod. But it looks like they want you to do the harder problem.
 
  • Like
Likes JulienB
  • #6
@TSny Thank you very much for your answer. I've given my homework back and am waiting for a feedback now. There was also an error in the script of my teacher for the Lorentz transformation of the electric field, that's why I was getting such a complicated equation. I'll let you know what the solution was once I get my homework back.
 
  • #7
JulienB said:
@TSny Thank you very much for your answer. I've given my homework back and am waiting for a feedback now. There was also an error in the script of my teacher for the Lorentz transformation of the electric field, that's why I was getting such a complicated equation. I'll let you know what the solution was once I get my homework back.
OK. Thanks.
 

1. What are relativistic E and B fields?

The relativistic E and B fields refer to the electric and magnetic fields, respectively, that are created by a moving charged particle. These fields are described by the theory of relativity, which takes into account the effects of motion and gravity.

2. How are the E and B fields related to an infinitely long wire?

An infinitely long wire is a theoretical concept that is often used to simplify calculations involving the E and B fields. In this case, the wire is considered to have a uniform charge density, and the E and B fields are perpendicular to each other and the direction of the wire.

3. How do the E and B fields change as the wire's speed approaches the speed of light?

As the wire's speed approaches the speed of light, the E and B fields become more and more distorted. This is due to the effects of length contraction and time dilation, which are predicted by the theory of relativity.

4. What is the significance of the E and B fields of an infinitely long wire?

The E and B fields of an infinitely long wire have significant implications in the field of electromagnetism. They are used to explain a wide range of phenomena, such as the behavior of charged particles in a magnetic field, the propagation of electromagnetic waves, and the behavior of electric and magnetic fields in materials.

5. Are there any practical applications of the relativistic E and B fields?

Yes, there are several practical applications of the relativistic E and B fields. These include the development of high-speed particle accelerators, the design of magnetic resonance imaging (MRI) machines, and the understanding of the behavior of charged particles in space, such as in the Earth's magnetic field.

Similar threads

  • Advanced Physics Homework Help
Replies
3
Views
399
  • Advanced Physics Homework Help
Replies
6
Views
314
  • Advanced Physics Homework Help
Replies
13
Views
2K
  • Advanced Physics Homework Help
Replies
1
Views
428
  • Advanced Physics Homework Help
Replies
3
Views
865
  • Advanced Physics Homework Help
Replies
2
Views
1K
  • Advanced Physics Homework Help
Replies
1
Views
591
  • Advanced Physics Homework Help
Replies
1
Views
1K
  • Advanced Physics Homework Help
Replies
3
Views
1K
  • Advanced Physics Homework Help
Replies
2
Views
1K
Back
Top